2025 AMC 12A Problems/Problem 22
Problem 22
Three real numbers are chosen independently and uniformly at random between
and
. What is the probability that the greatest of these three numbers is greater than
times each of the other two numbers? (In other words, if the chosen numbers are
, then
.)
Problem 22 (Chinese exams)
Three real numbers are chosen independently and uniformly at random between
and
. What is the probability that the greatest of these three numbers is greater than the sum of the other two numbers?
Solution 1
We can solve the problem by approaching it geometrically by mapping each possible triple to a coordinate
in a unit cube. Now, we just have to find the volume of the solution set over
.
WLOG, assume that
is the greatest number. The intersection between
and the unit cube is an oblique square pyramid with apex
and base vertices
. It helps to visualize the two planes cutting into the cube and leaving triangular traces in the
and
planes. The square base
, and the height
, so the volume
.
From here, we can multiply this volume by
to account for
being the greatest (all mutually exclusive). Alternatively, we can find
by taking
over the volume of the interesction between
and the unit cube (a pyramid wth base
and height
). Either way, we get the final probability of
.
~Zak2155
~Revised by Zhixing
Here is a desmos version created for a clear overview: https://www.desmos.com/3d/eidrbnqnvz ~DRA777
Solution 1.1 (same logic but faster)
Call the numbers
,
, and
for parity with the coordinate plane.
For simplicity, assume that
. This has a
chance of happening, so in the end we must remember to multiply our answer by
.
From here, we can use geometric probability. All possible triples
lie in a unit cube centered at
, which has a volume of
. However, our desired region is where
. We can graph this and find that the desired triples lie in a right tetrahedron of sides
,
, and
, Using
, the volume of our desired region is
. From this, we can see that the probability that we land in this region is
.
But remember that we still have to multiply our answer by 6, resulting in
.
~mikeypoo5608
Solution 2 (Integration)
Consider the bounds of
,
, and
. We know that
and
. For
to be in
,
must be in
. As such, each of
,
, and
must be in the following intervals:
\begin{align*}
1\geq &a\geq 2b, \\
0.5\geq &b\geq 0, \\
b\geq &c\geq 0.
\end{align*}
Now, we can simply integrate over these bounds:
Since there are
equally likely permutations of
,
, and
, we multiply
to obtain our answer of
Note that this is almost the same solution as Solution 1, where we find the volume of a similar polyhedron using a different method (except we do not need to visualize it). This method would remain effective even if the bounds were non-linear or if there were more than three variables.
Solution 3 (Logic)
Say that, WLOG,
is the maximum. Then notice that we need
, given that
, and similar for
. This probability is just
for
to be less, and
for
to be less (note that this is independent of the value of
). Therefore, the answer is
(technically, the answer is
but this is constant).
~ScoutViolet
Solution 4 (Chinese exams)
Let the three numbers be
,
, and
. By symmetry, the probability that we want to find is exactly
Taking the cube
in a Cartesian plane, we see that the region that corresponds to the event
is the set of points
Let
,
, and
, then the set above is equivalent to the set
It is then obvious that the solid given by this set is a tetrahedron formed by cutting the first octant with the plane
. The volume of it can be found by integration:
Hence, the answer is
Solution 5
Consider three independent uniform random variables
on
. The goal is to find the probability that the maximum value is greater than twice each of the other two values. Equivalently, if we order them as
, this is the probability that
(which implies
since
).
Due to symmetry, compute the probability that
is the maximum and satisfies the condition, then multiply by 3. The condition simplifies to
, as this ensures
is the maximum (almost surely).
The joint density is 1 over the unit cube. Thus,
\begin{align*}
P(X > 2Y \cap X > 2Z) &= \int_0^1 \int_0^{x/2} \int_0^{x/2} \, dy \, dz \, dx \\
&= \int_0^1 \left(\frac{x}{2}\right)^2 \, dx \\
&= \int_0^1 \frac{x^2}{4} \, dx \\
&= \frac{1}{4} \cdot \frac{1}{3} \\
&= \frac{1}{12}.
\end{align*}The total probability is
.
Alternatively, using order statistics
with joint density 6 on
,
\begin{align*}
P(W > 2V) &= \int_0^{0.5} \int_{2v}^1 \int_0^v 6 \, du \, dw \, dv \\
&= 6 \int_0^{0.5} v (1 - 2v) \, dv \\
&= 6 \left[ \frac{v^2}{2} - \frac{2v^3}{3} \right]_0^{0.5} \\
&= 6 \times \dfrac{1}{24} \\
&= \dfrac{1}{4}.
\end{align*}
No matter what entry points on this problem we choose, we can always get the correct answer: \(\color{red}\boxed{\color{black}\text{(E) } \frac14}\color{black}\).
~funkCCP
Solution 6
We have \(a,b,c \sim U(0,1)\). Thus, if \(a\) is the greatest, we must have \(b,c < a/2\), which occurs with probability \(a^2/4\). We integrate this over all \(a\) (note the pdf of \(U(0,1)\) is just \(1\)):
Since there are \(3\) possibilities for the greatest among \(a,b\), and \(c\), we multiply this by \(3\) to get \(3\cdot 1/12=\boxed{\text{(E) } \frac14}\)
~Jackson La Vallee
Video Solution 1 by OmegaLearn
Video Solution 2 by SpreadTheMathLove
https://www.youtube.com/watch?v=8wNoOAqd5_o
See Also
| 2025 AMC 12A (Problems • Answer Key • Resources) | |
| Preceded by Problem 21 |
Followed by Problem 23 |
| 1 • 2 • 3 • 4 • 5 • 6 • 7 • 8 • 9 • 10 • 11 • 12 • 13 • 14 • 15 • 16 • 17 • 18 • 19 • 20 • 21 • 22 • 23 • 24 • 25 | |
| All AMC 12 Problems and Solutions | |
These problems are copyrighted © by the Mathematical Association of America.